5
$\begingroup$

I am looking for the asymptotic growth of product of consecutive primes. Is there anything that is known about this growth?

$\endgroup$

3 Answers 3

7
$\begingroup$

Denote by $$\Pi(x)=\prod_{p\leqslant x}p,$$ thus $$\log\Pi(x)=\sum_{p\leqslant x}\log p:=\theta(x)\sim x,$$ which is known as the Prime Number Theorem. You may find further information in http://en.wikipedia.org/wiki/Prime_number_theorem

$\endgroup$
12
$\begingroup$

I think you're asking about the primorial function $n\sharp$, the product of all the primes less than or equal to $n$. This satisfies $ n \sharp = \exp( n(1+o(1)) ) $.

$\endgroup$
8
  • $\begingroup$ Are sharper bounds known? This is actually a pretty bad estimate for n small (less than, say, a billion). $\endgroup$
    – Charles
    Nov 1, 2010 at 16:21
  • $\begingroup$ @Charles: If you assume the Riemann Hypothesis, you get roughly $\exp(n+O(\sqrt{n}))$ $\endgroup$
    – S. Carnahan
    Nov 2, 2010 at 3:20
  • $\begingroup$ @Charles: Why do you say it is bad? Up to the (very) small bound of 350000 I find results such as [ 82619, .99728] [119549, .99537] [155893, .99855] [302831, .99671] [338477, .99898] meaning that at the prime n=82619 it is about $\exp( 0.99728n )$. etc. After a relatively large prime gap the exponent will be lower and after a relatively prime dense interval it will be higher. $\endgroup$ Nov 2, 2010 at 4:35
  • $\begingroup$ @Aaron Meyerowitz: exp(1e6)/1e6# is about 2e658. It's not so much that I mind being off by a factor of googol^6, but I wanted to know if more asymptotic terms were known. $\endgroup$
    – Charles
    Nov 2, 2010 at 5:07
  • $\begingroup$ @Charles OK so you are telling me that $10^6 \sharp$ is about $exp(10^6\cdot 0.99934)$. Working from that, I then get [1090697, .9991] ,[1195247, 1.00026] and [1243337, .99948]. I can't vouch for all those decimal places but the relative fluctuations should be pretty accurate. I can't imagine that asymptotic terms are going to account for fluctuations like that. The best you should expect is to get $\ln(n \sharp)$ about right and I think the answers above do that. $\endgroup$ Nov 2, 2010 at 7:24
5
$\begingroup$

You can also prove that $$\displaystyle\lim_n\left(\prod_1^n p_i\right)^{1/p_n} = e$$

(where $p_i$ is the $i$-th prime number and $e$ is Euler's exponential number)

$\endgroup$

Your Answer

By clicking “Post Your Answer”, you agree to our terms of service and acknowledge you have read our privacy policy.

Not the answer you're looking for? Browse other questions tagged or ask your own question.